Assumption

This topic has expert replies
User avatar
Master | Next Rank: 500 Posts
Posts: 351
Joined: Mon Jul 04, 2011 10:25 pm
Thanked: 57 times
Followed by:4 members

Assumption

by akhilsuhag » Tue Aug 12, 2014 12:57 pm
McCray's Dairy Industries stands to reap tremendous profits in the coming year. The new milking apparatus and packing equipment purchased and installed last year allows McCray's to produce the same quantity of dairy products at less than one third of the previous cost. Restaurants and academic institutions throughout the state will choose to purchase all their dairy products from McCray's, because they will be available at a much lower cost than similar products from rival producers.

The argument above assumes which of the following?

(A) Statewide demand for dairy products will increase substantially in the coming year

(B) Buyers at restaurants and academic institutions throughout the state would prefer McCray's dairy products over rival dairy products if cost were no object

(C)Physicians see no reason to predict a sharp increase in the number of dairy allergies throughout the state.

(D) Statewide production of dairy products will not increase in the coming year.

(E) The quality of McCray's dairy products meets the acceptability standards of most restaurants and academic institutions throughout the state.
Please press "thanks" if you think my post has helped you.. Cheers!!

User avatar
Senior | Next Rank: 100 Posts
Posts: 95
Joined: Sat Dec 28, 2013 8:53 am
Location: United States
Thanked: 2 times
Followed by:5 members

by tanvis1120 » Tue Aug 12, 2014 6:32 pm
akhilsuhag wrote:McCray's Dairy Industries stands to reap tremendous profits in the coming year. The new milking apparatus and packing equipment purchased and installed last year allows McCray's to produce the same quantity of dairy products at less than one third of the previous cost. Restaurants and academic institutions throughout the state will choose to purchase all their dairy products from McCray's, because they will be available at a much lower cost than similar products from rival producers.

The argument above assumes which of the following?

(A) Statewide demand for dairy products will increase substantially in the coming year

(B) Buyers at restaurants and academic institutions throughout the state would prefer McCray's dairy products over rival dairy products if cost were no object

(C)Physicians see no reason to predict a sharp increase in the number of dairy allergies throughout the state.

(D) Statewide production of dairy products will not increase in the coming year.

(E) The quality of McCray's dairy products meets the acceptability standards of most restaurants and academic institutions throughout the state.
Is the OA: E? Please let me know.

A. Even if the statewide demand increases, it does not explain why McCray's would be preferred.
B. Attacks the conclusion. Conclusion says that McCray's will available at lower cost and consumers will prefer to buy their products.
C.Can be a possible assumption (Kept aside)
D. I could not make out what to do with it. So, eliminated.
E. Strong enough to be an assumption. Even if the prices of McCray;s diary products are lower, people would not buy their products if the quality is below average / poor. So, it is assumed that McCray's dairy products meet the quality criteria.

@Experts: please correct me if I am wrong.

User avatar
Master | Next Rank: 500 Posts
Posts: 351
Joined: Mon Jul 04, 2011 10:25 pm
Thanked: 57 times
Followed by:4 members

by akhilsuhag » Wed Aug 13, 2014 3:15 am
Hey,

Yes the OA is E

I although have my doubts about B, that is why I posted this question. Paraphrasing the prompt:

"Big profits expected as they will reduce costs -> Consumers will buy them because of price superiority compared to similar products"

B says Cost is a major reason they dont buy McCray. If that was not the case then even with decresed costs no profits will come McCray's way.

I might be over thinking this, but this seems to be a more direct assumption than E to me.
Please press "thanks" if you think my post has helped you.. Cheers!!

Newbie | Next Rank: 10 Posts
Posts: 8
Joined: Mon May 07, 2012 10:15 am

by mdlyman » Wed Aug 13, 2014 7:56 am
I'd agree that the answer is E.

User avatar
Senior | Next Rank: 100 Posts
Posts: 95
Joined: Sat Dec 28, 2013 8:53 am
Location: United States
Thanked: 2 times
Followed by:5 members

by tanvis1120 » Wed Aug 13, 2014 8:43 am
akhilsuhag wrote:Hey,

Yes the OA is E

I although have my doubts about B, that is why I posted this question. Paraphrasing the prompt:

"Big profits expected as they will reduce costs -> Consumers will buy them because of price superiority compared to similar products"

B says Cost is a major reason they dont buy McCray. If that was not the case then even with decresed costs no profits will come McCray's way.

I might be over thinking this, but this seems to be a more direct assumption than E to me.
Hi Akhil,

What is my perception of B?

If cost were no object, reducing the price of products will not affect McCray's profitability . I hope, it makes a little clear..

Thanks,
TS